Sie sind auf Seite 1von 4

MTH 102: Linear Algebra

Department of Mathematics and Statistics Indian Institute of Technology - Kanpur

Problem Set 4

Problems marked (T) are for discussions in Tutorial sessions.

1. Determine whether the following sets of vectors are linearly independent or not

(a) {(1, 0, 0), (1, 1, 0), (1, 1, 1)} of R3


 
1 1 1
Solution: Yes. Look at the null space, N (A) of A =  0 1 1 . One can show that
0 0 1
N (A) = {0} by computing the reduced row echelon form, R.
(b) {(1, 0, 0, 0), (1, 1, 0, 0), (1, 2, 0, 0), (1, 1, 1, 1)} of R4
   
1 1 1 1 −1
 0 1 2 1 
  2 
Solution: No. The null space N (A) of A =   0 contains  .
0 0 1   −1 
0 0 0 1 0
(c) {(1, 0, 2, 1), (1, 3, 2, 1), (4, 1, 2, 2)} in R4 .
Solution: Yes. Similar to (a).

2. Find a maximal linearly independent subset of


               
 1 −1 0 1 0 2 2 −1 
 2   0  2 −1 0 1 3 −2

 

S = −1,  1 , 2, −1, 0, 1, 0, −1 .
               

 0

    1  2 −3 2 1 1  0  
 
1 2 1 1 1 0 1 1

Find another. And another. Do they have the same cardinality?

3. Give 2 bases for the trace 0 real symmetric matrices of size 3 × 3. Extend these bases to bases
of the real matrices of size 3 × 3.

4. Consider W = {v ∈ R6 : v1 + v2 + v3 = 0, v2 + v3 + v4 = 0, v5 + v6 = 0}. Supply a basis for W


and extend it to a basis of R6 .
   
1 0 0 0
5. Let M be the vector space of all 2 × 2 matrices and let A = ,B = .
0 0 0 −1

(a) Give a basis of M .


       
1 0 0 1 0 0 0 0
Solution: One basis would be , , , .
0 0 0 0 1 0 0 1
(b) Describe a subspace of M which contains A and does not contain B.
Solution: The subspace consisting of all multiples of A is a subspace which contains A
but not B.
2

(c) True (give a reason) or False (give a counter example) : If a subspace of M contains A and
B, it must contain the identity matrix.
Solution: True : If a subspace contains A and B then it also contains A − B = I.
6. [T] Let {w1 , w2 , . . . , wn } be a basis of the finite dimensional vector space V. Let v be any non
zero vector in V. Show that there exists wi such that if we replace wi by v then we still have a
basis.

loss of generality, we can assume that v = ni=1 αi wi with


P
Solution: Since v 6= 0, without
α1 6= 0. So, w1 = α11 v − α11 ni=2
P
Pα i wi . Thus {v, w2 , . . . , wn } also spans V. Now, β1 v + β2 w2 +
n
· · · + βn wn = 0 ⇒ β1 α1 w1 + i=2 (β1 αi + βi )wi = 0 As {w1 , w2 , . . . , wn } is a basis, we get
β1 α1 = 0 ⇒ β1 = 0 (α1 6= 0) ⇒ βi = 0, i ≥ 2.
7. Show that {u, v} is linearly independent if and only if {u + v, u − v} is linearly independent.
8. (T) Show that u1 , . . . , uk ∈ Rn are linearly independent if and only if Au1 , . . . , Auk are linearly
independent for any invertible matrix An×n .
That is, suppose we have an n × n invertible matrix A and consider the map f : Rn → Rn
defined by f (x) = Ax. Then, ‘u1 , . . . , uk are linearly independent if and only if their images are
also linearly independent’.
P
Solution: SupposeP u1 , . . .P, un are linearly dependent. Then there exists α 6= 0 s.t. αi ui = 0.
So 0 = A0 = A αi ui = αi (Aui ). Hence, Au1 , . . . , Aun are linearly dependent.
P
Now, supposeP that Au1 , . . . ,P
Aun are linearly dependent. ThenP there exists
P α 6= 0 s.t. αi (Aui ) =
0. So 0 = αi (Aui ) = A αi ui . Hence, A−1 0 = A−1 A αi ui = αi ui . Thus, u1 , . . . , un
are linearly dependent.
k
P k
P
9. Show that u1 , . . . , uk ∈ V is linearly independent if and only if ai1 ui , . . . , aik ui are linearly
i=1 i=1
independent for any invertible matrix Ak×k . This means: In LS(u1 , . . . , uk ) the set {u1 , . . . , uk }
Pk
are linearly independent if and only if the vectors wj = aij ui (which are nothing but some
i=1
linear combinations of ui ’s given by the matrix A) are linearly independent.
w1 a11 · · · ak1 u1 u1
      
k
P ..  .. .. .. t ..
Solution: Put wr = air ui . Then  . = . . . =A
 . .
i=1
wr a1k ··· akk uk uk
w1
 

Suppose w1 , . . . , wk are linearly dependent. Then there exists α 6= 0 s.t. [α1 · · · αk ] ... = 0.
wk
So
a11 · · · ak1 u1 u1
    

0 = [α1 · · · αk ] ... .. ..


. . = [β1 ··· βk ] ...,
a1k ··· akk uk uk
where [α1 · · · αk ]At = [β1 ··· βk ] 6= 0. Thus u1 , . . . , uk are linearly dependent.
Converse: Similar.
3

10. (T) If v1 , v2 , . . . , vd is a basis for a vector space V, then show that any set of n vectors in V
with n > d, say {w1 , w2 , . . . , wn }, is linearly dependent.

Solution: Since v1 , v2 , . . . , vd is a basis for V and wj ∈ V for j = 1, . . . , n, there exist constants


aij , 1 ≤ i ≤ d, 1 ≤ j ≤ n such that
Xd
wj = aij vi .
i=1
Pn
Consider a linear combination of wj ’s that equals zero, that is, j=1 cj wj = 0. Then,
!  
n
X n
X d
X d
X X n
cj wj = 0 ⇐⇒ cj aij vi = 0 ⇐⇒  aij cj  vi = 0.
j=1 j=1 i=1 i=1 j=1

As vi ’s are linearly independent, we have Ac = 0 where the matrix A is a d by n matrix and c


is a column vector of size n with [A]ij = aij . As A is a rectangular matrix with
Pmore columns
than rows, its null space is non-trivial. We therefore have non-zero cj ’s with nj=1 cj wj = 0.
Thus, vectors w1 , w2 , . . . , wn are linearly dependent.
11. Suppose V is a vector space of dimension d. Let S = {w1 , w2 , . . . , wn } be a set of vectors from
V. Then show that S does not span V if n < d.

Solution: Let B be a basis of V . Since dim(V) = d, the definition imply that B is a linearly
independent set of d vectors that spans V.
Now, suppose on the contrary that S does span V. Then B is a larger set of vectors that is
linearly independent. This contradicts the result in the previous problem.
12. (T) Determine if the set T = {1, x2 − x + 5, 4x3 − x2 + 5x, 3x + 2} spans the vector space of
polynomials with degree 4 or less.

Solution: The vector space R[x; 4] has dimension 5. Since T contains only 3 vectors, T does
not span R[x; 4]. But, do check that it forms a basis of R[x; 3].
13. Let W be a proper subspace of V.
(a) Show that there is a subspace U of V such that W ∩ U = {0} and U + W = V.
Solution: Extend the basis of W to a basis of V and define U to be the span of new basis
elements.
(b) Show that there is no subspace U such that U ∩ W = {0} and dim U + dim W > dim V.
Solution: Follows from dim(U + W) = dim(U) + dim(W) − dim(U ∩ W) (just ask the
students to assume this result) and the fact that U + W is a subspace of V.
14. (T) Describe all possible ways in which two planes (passing through origin) in R3 could intersect.

Solution: Let U and V be planes. Then, dim(U + V) = dim(U) + dim(V) − dim(U ∩ V) implies
that dim(U + V) = 4 − dim(U ∩ V). Clearly, 2 ≤ dim(U + V) ≤ 3. If dim(U + V) = 2, then
dim(U ∩ V) = 2 which implies U + V = U = V = U ∩ V, i.e., U = V. If dim(U + V) = 3, then
dim(U ∩ V) = 1 which implies that U and V intersect on a line.
4

15. Construct a matrix with the required property or explain why this is impossible:
   
1 0    
1 2
(a) Column space contains 1, 0, row space contains , .
2 5
0 1
 
1 0
Solution: 1 0.

0 1
     
 1   3  3
(b) Column space has basis  1  , null-space has basis  1  . What if 1 belongs to

3 1 1
   
the null space (but not necessarily forms a basis)?
Solution: Not possible; dimension of the column  space andthe dimension of the null-space
1 1 −4
must add to 3. For the second part, take A = 1 1 −4  .
3 3 −12
(c) The dimension of null-space is one more than the dimension of left null-space.
 
  1 1 2
Solution: 1 1 or
1 1 2
   
1 3
(d) Left null-space contains , row space contains .
3 1
 
-9 -3
Solution: .
3 1
16. Suppose A is a 3 by 4 matrix and B is a 4 by 5 matrix with AB = 0. Show that
rank(A) + rank(B) ≤ 4.

Solution: As AB = 0, col(B) ⊆ N (A). Therefore, the dim(col(B)) ≤ dim(N (A)). This implies
rank(B) ≤ 4 − rank(A).
17. (T) Let A be an m by n matrix and B be an n by p matrix with rank(A) = rank(B) = n. Show
that rank(AB) = n.

Solution: We will use


• rank(AB) ≤ rank(A) for any two matrices A and B.
• If B is an invertible matrix then rank(AB) = rank(A).
Now, note that rank(AB) ≤ rank(A) = n. Also, BB t is an n × n matrix of rank n and hence is
invertible. So, rank(ABB t ) = rank(A). Thus,
n = rank(A) = rank(ABB t ) ≤ rank(AB).
Hence, the required result follows.

Das könnte Ihnen auch gefallen